LSAT and Law School Admissions Forum

Get expert LSAT preparation and law school admissions advice from PowerScore Test Preparation.

User avatar
 Dave Killoran
PowerScore Staff
  • PowerScore Staff
  • Posts: 5853
  • Joined: Mar 25, 2011
|
#28014
Complete Question Explanation
(The complete setup for this game can be found here: lsat/viewtopic.php?t=11796)

The correct answer choice is (E)

The question stem states that there must be exactly three colors in the costume. Templates #2 and #3 feature exactly two colors, and therefore template #1 as shown in the setup discussion is the template that applies to this question. The question stem also references the overalls, and since in template #1 the overalls are either red or violet, the correct answer must indicate that the overalls are red or violet, or indicate that the overalls are not yellow.

Answer choice (A): This answer choice is incorrect. In template #1, the overalls cannot be plaid.

Answer choice (B): This answer choice is incorrect. In template #1, the overalls cannot be plaid (or yellow, for that matter).

Answer choice (C): This answer choice is incorrect. In template #1, the overalls cannot be plaid.

Answer choice (D): This answer choice is incorrect. In template #1, the overalls cannot be yellow.

Answer choice (E): This is the correct answer choice.


Overall, this game is relatively easy as long as you use the numerical distribution to produce templates, which should enable you to complete the game very quickly with perfect accuracy.
User avatar
 Mmjd12
  • Posts: 34
  • Joined: Apr 12, 2023
|
#103616
Is this an example of the Overlap Principle? I got this correct I am just curious if I'm understanding the overlap principle correctly.
 Rachael Wilkenfeld
PowerScore Staff
  • PowerScore Staff
  • Posts: 1358
  • Joined: Dec 15, 2011
|
#103716
Yes, MMJD. The idea here is that there are three total colors over the four total slots for J and O in this example. If O was plaid, then there would only be two colors over the three slots including the overlap between the J and the O. So in order to have three total colors, we need J to be plaid, and O to be single color, a single color which also occurs in J.

Good work!

Get the most out of your LSAT Prep Plus subscription.

Analyze and track your performance with our Testing and Analytics Package.